Question and Answers Forum

All Questions      Topic List

Others Questions

Previous in All Question      Next in All Question      

Previous in Others      Next in Others      

Question Number 96764 by Rio Michael last updated on 04/Jun/20

A particle P of mass m, is projected vertically upward with  a speed u from a point A, on horizontal ground. When P is at x above  its initial position, its speed is v. The only forces acting on P is  its weight and resistance mgkv^2 . where k is a positive constant.  (a) Show that the greatest height reached is (1/(2gk)) ln(1 +ku^2 ).  (b) show that the speed with which P returns to A is (u/(√(1+ ku^2 ))) .

$$\mathrm{A}\:\mathrm{particle}\:\mathrm{P}\:\mathrm{of}\:\mathrm{mass}\:{m},\:\mathrm{is}\:\mathrm{projected}\:\mathrm{vertically}\:\mathrm{upward}\:\mathrm{with} \\ $$$$\mathrm{a}\:\mathrm{speed}\:{u}\:\mathrm{from}\:\mathrm{a}\:\mathrm{point}\:{A},\:\mathrm{on}\:\mathrm{horizontal}\:\mathrm{ground}.\:\mathrm{When}\:\mathrm{P}\:\mathrm{is}\:\mathrm{at}\:{x}\:\mathrm{above} \\ $$$$\mathrm{its}\:\mathrm{initial}\:\mathrm{position},\:\mathrm{its}\:\mathrm{speed}\:\mathrm{is}\:{v}.\:\mathrm{The}\:\mathrm{only}\:\mathrm{forces}\:\mathrm{acting}\:\mathrm{on}\:\mathrm{P}\:\mathrm{is} \\ $$$$\mathrm{its}\:\mathrm{weight}\:\mathrm{and}\:\mathrm{resistance}\:{m}\mathrm{g}{kv}^{\mathrm{2}} .\:\mathrm{where}\:{k}\:\mathrm{is}\:\mathrm{a}\:\mathrm{positive}\:\mathrm{constant}. \\ $$$$\left(\mathrm{a}\right)\:\mathrm{Show}\:\mathrm{that}\:\mathrm{the}\:\mathrm{greatest}\:\mathrm{height}\:\mathrm{reached}\:\mathrm{is}\:\frac{\mathrm{1}}{\mathrm{2g}{k}}\:\mathrm{ln}\left(\mathrm{1}\:+{ku}^{\mathrm{2}} \right). \\ $$$$\left(\mathrm{b}\right)\:\mathrm{show}\:\mathrm{that}\:\mathrm{the}\:\mathrm{speed}\:\mathrm{with}\:\mathrm{which}\:\mathrm{P}\:\mathrm{returns}\:\mathrm{to}\:\mathrm{A}\:\mathrm{is}\:\frac{{u}}{\sqrt{\mathrm{1}+\:{ku}^{\mathrm{2}} }}\:. \\ $$

Commented by mr W last updated on 04/Jun/20

Commented by mr W last updated on 04/Jun/20

no further comment...

$${no}\:{further}\:{comment}... \\ $$

Commented by Rio Michael last updated on 05/Jun/20

sorry sir, but on my phone it doesn′t look that way.  its normal.

$$\mathrm{sorry}\:\mathrm{sir},\:\mathrm{but}\:\mathrm{on}\:\mathrm{my}\:\mathrm{phone}\:\mathrm{it}\:\mathrm{doesn}'\mathrm{t}\:\mathrm{look}\:\mathrm{that}\:\mathrm{way}. \\ $$$$\mathrm{its}\:\mathrm{normal}. \\ $$

Commented by mr W last updated on 06/Jun/20

you have a large screen smart phone.  then it′s not possible for you to  control the line length while typing.

$${you}\:{have}\:{a}\:{large}\:{screen}\:{smart}\:{phone}. \\ $$$${then}\:{it}'{s}\:{not}\:{possible}\:{for}\:{you}\:{to} \\ $$$${control}\:{the}\:{line}\:{length}\:{while}\:{typing}. \\ $$

Commented by Rio Michael last updated on 08/Jun/20

i′ll try decreasing it. thanks

$$\mathrm{i}'\mathrm{ll}\:\mathrm{try}\:\mathrm{decreasing}\:\mathrm{it}.\:\mathrm{thanks} \\ $$

Answered by mr W last updated on 04/Jun/20

upwards:  ma=−mg−mgkv^2   a=(dv/dt)=−g(1+kv^2 )  v(dv/dh)=−g(1+kv^2 )  ((2vkdv)/(1+kv^2 ))=−2gkdh  ((d(1+kv^2 ))/(1+kv^2 ))=−2gkdh  ∫_u ^0 ((d(1+kv^2 ))/(1+kv^2 ))=−2gk∫_0 ^h_(max)  dh  [ln (1+kv^2 )]_u ^0 =−2gk[h]_0 ^h_(max)    ⇒h_(max) =(1/(2gk))ln (1+ku^2 )    downwards:  ma=mg−mgkv^2   a=(dv/dt)=g(1−kv^2 )  −v(dv/dh)=g(1−kv^2 )  ((−2vkdv)/(1−kv^2 ))=2gkdh  ((d(1−kv^2 ))/(1−kv^2 ))=2gkdh  ∫_0 ^v_A  ((d(1−kv^2 ))/(1−kv^2 ))=2gk∫_h_(max)  ^0 dh  [ln (1−kv^2 )]_0 ^v_A  =2gk[h]_h_(max)  ^0   ln (1−kv_A ^2 )=−2gkh_(max)   ln (1−kv_A ^2 )=−ln (1+ku^2 )  1−kv_A ^2 =(1/(1+ku^2 ))  v_A ^2 =(u^2 /(1+ku^2 ))  ⇒v_A =(u/(√(1+ku^2 )))

$${upwards}: \\ $$$${ma}=−{mg}−{mgkv}^{\mathrm{2}} \\ $$$${a}=\frac{{dv}}{{dt}}=−{g}\left(\mathrm{1}+{kv}^{\mathrm{2}} \right) \\ $$$${v}\frac{{dv}}{{dh}}=−{g}\left(\mathrm{1}+{kv}^{\mathrm{2}} \right) \\ $$$$\frac{\mathrm{2}{vkdv}}{\mathrm{1}+{kv}^{\mathrm{2}} }=−\mathrm{2}{gkdh} \\ $$$$\frac{{d}\left(\mathrm{1}+{kv}^{\mathrm{2}} \right)}{\mathrm{1}+{kv}^{\mathrm{2}} }=−\mathrm{2}{gkdh} \\ $$$$\int_{{u}} ^{\mathrm{0}} \frac{{d}\left(\mathrm{1}+{kv}^{\mathrm{2}} \right)}{\mathrm{1}+{kv}^{\mathrm{2}} }=−\mathrm{2}{gk}\int_{\mathrm{0}} ^{{h}_{{max}} } {dh} \\ $$$$\left[\mathrm{ln}\:\left(\mathrm{1}+{kv}^{\mathrm{2}} \right)\right]_{{u}} ^{\mathrm{0}} =−\mathrm{2}{gk}\left[{h}\right]_{\mathrm{0}} ^{{h}_{{max}} } \\ $$$$\Rightarrow{h}_{{max}} =\frac{\mathrm{1}}{\mathrm{2}{gk}}\mathrm{ln}\:\left(\mathrm{1}+{ku}^{\mathrm{2}} \right) \\ $$$$ \\ $$$${downwards}: \\ $$$${ma}={mg}−{mgkv}^{\mathrm{2}} \\ $$$${a}=\frac{{dv}}{{dt}}={g}\left(\mathrm{1}−{kv}^{\mathrm{2}} \right) \\ $$$$−{v}\frac{{dv}}{{dh}}={g}\left(\mathrm{1}−{kv}^{\mathrm{2}} \right) \\ $$$$\frac{−\mathrm{2}{vkdv}}{\mathrm{1}−{kv}^{\mathrm{2}} }=\mathrm{2}{gkdh} \\ $$$$\frac{{d}\left(\mathrm{1}−{kv}^{\mathrm{2}} \right)}{\mathrm{1}−{kv}^{\mathrm{2}} }=\mathrm{2}{gkdh} \\ $$$$\int_{\mathrm{0}} ^{{v}_{{A}} } \frac{{d}\left(\mathrm{1}−{kv}^{\mathrm{2}} \right)}{\mathrm{1}−{kv}^{\mathrm{2}} }=\mathrm{2}{gk}\int_{{h}_{{max}} } ^{\mathrm{0}} {dh} \\ $$$$\left[\mathrm{ln}\:\left(\mathrm{1}−{kv}^{\mathrm{2}} \right)\right]_{\mathrm{0}} ^{{v}_{{A}} } =\mathrm{2}{gk}\left[{h}\right]_{{h}_{{max}} } ^{\mathrm{0}} \\ $$$$\mathrm{ln}\:\left(\mathrm{1}−{kv}_{{A}} ^{\mathrm{2}} \right)=−\mathrm{2}{gkh}_{{max}} \\ $$$$\mathrm{ln}\:\left(\mathrm{1}−{kv}_{{A}} ^{\mathrm{2}} \right)=−\mathrm{ln}\:\left(\mathrm{1}+{ku}^{\mathrm{2}} \right) \\ $$$$\mathrm{1}−{kv}_{{A}} ^{\mathrm{2}} =\frac{\mathrm{1}}{\mathrm{1}+{ku}^{\mathrm{2}} } \\ $$$${v}_{{A}} ^{\mathrm{2}} =\frac{{u}^{\mathrm{2}} }{\mathrm{1}+{ku}^{\mathrm{2}} } \\ $$$$\Rightarrow{v}_{{A}} =\frac{{u}}{\sqrt{\mathrm{1}+{ku}^{\mathrm{2}} }} \\ $$

Commented by peter frank last updated on 04/Jun/20

thank you

$$\mathrm{thank}\:\mathrm{you} \\ $$

Commented by Rio Michael last updated on 05/Jun/20

thank you sir.

$$\mathrm{thank}\:\mathrm{you}\:\mathrm{sir}. \\ $$

Terms of Service

Privacy Policy

Contact: info@tinkutara.com